Đề bài

Bất đẳng thức nào sau đây đúng với mọi \(a > 0,b > 0:\)

  • A.

    \({a^3} + {b^3} \le a{b^2} + {a^2}b\)

  • B.

    \({a^3} + {b^3} \ge a{b^2} + {a^2}b\)

  • C.

    \(a{b^2} + {a^2}b = {a^3} + {b^3}\)

  • D.

    \(a{b^2} + {a^2}b > {a^3} + {b^3}\)

Phương pháp giải :

+ Xét hiệu \({a^3} + {b^3} - a{b^2} - {a^2}b\)

+ Phân tích vế trái thành nhân tử và đánh giá theo điều kiện của \(a,b\).

Lời giải chi tiết :

Ta có: \({a^3} + {b^3} - a{b^2} - {a^2}b = {a^2}(a - b) - {b^2}(a - b)\)

\( = {(a - b)^2}(a + b) \ge 0\) (vì \({\left( {a - b} \right)^2} \ge 0\) với mọi \(a,b\) và \(a + b > 0\) với \(a > 0,b > 0\)).

Do đó \({a^3} + {b^3} - a{b^2} - {a^2}b \ge 0\) hay \({a^3} + {b^3} \ge a{b^2} + {a^2}b\).

Đáp án : B

Các bài tập cùng chuyên đề

Bài 1 :

Hãy chọn câu sai:

Xem lời giải >>
Bài 2 :

Hãy chọn câu đúng. Nếu \(a > b\) thì:

Xem lời giải >>
Bài 3 :

Hãy chọn câu sai. Nếu \(a < b\) thì:

Xem lời giải >>
Bài 4 :

Cho \(a + 1 \le b + 2\). So sánh  $2$  số \(2a + 2\) và \(2b + 4\) nào dưới đây là đúng?

Xem lời giải >>
Bài 5 :

Cho \( - 2x + 3 <  - 2y + 3\). So sánh $x$  và $y$ . Đáp án nào sau đây là đúng?

Xem lời giải >>
Bài 6 :

Cho \(a > b > 0.\) So sánh \({a^2}\) và \(ab\); \({a^3}\) và \({b^3}\) .

Xem lời giải >>
Bài 7 :

Cho $a,b$ bất kì. Chọn câu đúng.

Xem lời giải >>
Bài 8 :

Cho \( - 2018a <  - 2018b\). Khi đó

Xem lời giải >>
Bài 9 :

Với mọi \(a,b,c\) . Khẳng định nào sau đây là đúng?

Xem lời giải >>
Bài 10 :

Cho \(x + y > 1.\) Chọn khẳng định đúng

Xem lời giải >>
Bài 11 :

Bất đẳng thức nào sau đây đúng với mọi \(a > 0,b > 0:\)

Xem lời giải >>
Bài 12 :

Cho \(a \ge b > 0\). Khẳng định nào đúng?

Xem lời giải >>
Bài 13 :

Cho \(x > 0;y > 0\). Tìm khẳng định đúng trong các khẳng định sau?

  \(\left( 1 \right)\;\;\;(x + y)\left( {\dfrac{1}{x} + \dfrac{1}{y}} \right) \ge 4\)                                           

 \(\left( 2 \right)\;\;\;\;{x^2} + {y^3} \le 0\)

\(\left( 3 \right)\;\;\;(x + y)\left( {\dfrac{1}{x} + \dfrac{1}{y}} \right) < 4\)

Xem lời giải >>
Bài 14 :

So sánh \(m\) và \({m^2}\) với \(0 < m < 1\) .

Xem lời giải >>